Difference between revisions of "2013 USAMO Problems/Problem 1"
(→Solution 3) |
(→Solution 1) |
||
Line 81: | Line 81: | ||
In this solution, all lengths and angles are directed. | In this solution, all lengths and angles are directed. | ||
− | Firstly, it is easy to see by that <math>\omega_A, \omega_B, \omega_C</math> concur at a point <math>M</math>. Let <math>XM</math> meet <math>\omega_B, \omega_C</math> again at <math>D</math> and <math>E</math>, respectively. Then by Power of a Point, we have <cmath>XM \cdot XE = XZ \cdot XP \quad\text{and}\quad XM \cdot XD = XY \cdot XP</cmath> Thusly <cmath>\frac{XY}{XZ} = \frac{XD}{XE}</cmath> But we claim that <math>\triangle XDP \sim \triangle PBM</math>. Indeed, <cmath>\measuredangle XDP = \measuredangle MDP = \measuredangle MBP = - \measuredangle PBM</cmath> and <cmath>\measuredangle DXP = \measuredangle MXY = \measuredangle MXA = \measuredangle MRA = \measuredangle MRB = \measuredangle MPB = -\measuredangle BPM</cmath> | + | Firstly, it is easy to see by that <math>\omega_A, \omega_B, \omega_C</math> concur at a point <math>M</math>. Let <math>XM</math> meet <math>\omega_B, \omega_C</math> again at <math>D</math> and <math>E</math>, respectively. Then by Power of a Point, we have <cmath>XM \cdot XE = XZ \cdot XP \quad\text{and}\quad XM \cdot XD = XY \cdot XP</cmath> Thusly <cmath>\frac{XY}{XZ} = \frac{XD}{XE}</cmath> But we claim that <math>\triangle XDP \sim \triangle PBM</math>. Indeed, <cmath>\measuredangle XDP = \measuredangle MDP = \measuredangle MBP = - \measuredangle PBM</cmath> and <cmath>\measuredangle DXP = \measuredangle MXY = \measuredangle MXA = \measuredangle MRA = 180^\circ - \measuredangle MRB = \measuredangle MPB = -\measuredangle BPM</cmath> |
Therefore, <math>\frac{XD}{XP} = \frac{PB}{PM}</math>. Analogously we find that <math>\frac{XE}{XP} = \frac{PC}{PM}</math> and we are done. | Therefore, <math>\frac{XD}{XP} = \frac{PB}{PM}</math>. Analogously we find that <math>\frac{XE}{XP} = \frac{PC}{PM}</math> and we are done. | ||
Line 87: | Line 87: | ||
courtesy v_enhance | courtesy v_enhance | ||
---- | ---- | ||
+ | |||
==Solution 2== | ==Solution 2== | ||
Revision as of 23:11, 31 July 2015
Contents
Problem
In triangle , points lie on sides respectively. Let , , denote the circumcircles of triangles , , , respectively. Given the fact that segment intersects , , again at respectively, prove that
Solution 1
In this solution, all lengths and angles are directed.
Firstly, it is easy to see by that concur at a point . Let meet again at and , respectively. Then by Power of a Point, we have Thusly But we claim that . Indeed, and Therefore, . Analogously we find that and we are done.
courtesy v_enhance
Solution 2
Diagram Refer to the Diagram link.
By Miquel's Theorem, there exists a point at which intersect. We denote this point by Now, we angle chase: In addition, we have Now, by the Ratio Lemma, we have (by the Law of Sines in ) (by the Law of Sines in ) by the Ratio Lemma. The proof is complete.
Solution 3
Use directed angles modulo .
Lemma.
Proof.
Now, it follows that (now not using directed angles) using the facts that and , and are similar triangles, and that equals twice the circumradius of the circumcircle of .
The problems on this page are copyrighted by the Mathematical Association of America's American Mathematics Competitions.